A company makes paper labels for paint cans. As shown below, each can is in the shape of a cylinder
with a height of 7 cm and a radius of 2 cm. The paper label is wrapped around the can and covers only
the side of the can (not the top or bottom). The company uses, on average, 3612.64 cm² of paper
each minute making these labels. How many labels does the company make, on average, each
minute?
For your calculations, do not round any intermediate steps, and use the button on a calculator. Round
your answer to the nearest hundredth.

A Company Makes Paper Labels For Paint Cans. As Shown Below, Each Can Is In The Shape Of A Cylinderwith

Answers

Answer 1

The company needs 41 labels on average, each minute.

What is the circumference of a circle?

The perimeter of a circle (circumference): 2 π radius

Replacing with the values given:

C = 2 (3.14) 2 = 12.56 cm

So, the area of one label is:

Area of a rectangle: length x width = 12.56 x 7 = 87.92 cm sq.

Now, the number of label be n

87.92 x n = 3612.64 cm2

n = 3612.64 / 87.92

n = 41.09

Hence, The company needs 41 labels on average, each minute.

Learn more about circumference;

https://brainly.com/question/4268218

#SPJ1


Related Questions

What is the period of this graph?

Answers

Answer:

Look at the pic for answer.

Step-by-step explanation:

Which of the function correctly expresses the exponential model f(x)=6(14)^x with base of e

Answers

The exponential equation with a base of e is:

[tex]f(x) = 6*e^{2.639*x}[/tex]

How to change the base?

We start with:

[tex]f(x) = 6*(14)^x[/tex]

We want to rewrite this to:

[tex]f(x) = 6*(e^n)^x[/tex]

So we only need to find the value of n such that:

[tex]e^n = 14[/tex]

If we apply the natural logarithm to both sides:

[tex]ln(e^n) = ln(14)\\n = ln(14) = 2.639[/tex]

Then the exponential equation with base of e is:

[tex]f(x) = 6*e^{2.639*x}[/tex]

If you want to learn more about exponentials:

https://brainly.com/question/11464095

#SPJ1

Joshua has a job that pays $2,265 biweekly. How much is withheld annually for Medicare? A.$853.91 B.$32.84 C.$16.42 d.$35.58 E.$71.16

Answers

Answer:

09999kjht

Step-by-step explanation:

tttttttttgtgr312345678909ytrewq

An 8-pack of clay pots costs $15.76. What is the unit price?

Answers

Here
Clay pots =8
Price =$15.76
Then unit price =15.6/8
=$1.95

Determine the number of crawlers and rovers to make,that will maximize the company profit in a given week

Answers

Answer:

jjjijijijiiijijijijijijij no pliss inglés. ;D.

Solve the following word problem.
Money is invested at two rates of interest. One rate is 8 % and the other is 2%. If there is $1300 more invested at 8 % than at 2 %. find the amount invested at
each rate if the total annual interest received is $350. Let x = amount invested at 8% and y = amount invested at 2 %. Then the system that models the problem
[x = y + 1300
is
Solve the system by using the method of addition.
0.08x +0.02y = 350,

Answers

Answer:

at 8%: $3760at 2%: $2460

Step-by-step explanation:

You are given a system of equations and asked to solve it by the method of addition. That method requires you add a multiple of one equation to the other so that one of the variables is eliminated. In some cases, this is easier if multiples of both equations are added together. The resulting single-variable equation is then solved in the usual way.

__

look

The given system of equations is ...

x = y +13000.08x +0.02y = 350

We notice the first equation has the variables on opposite sides of the equal sign, and both their coefficients are 1. The second equation has the variables on the same side of the equal sign, and their coefficients are 0.08 and 0.02.

plan

To eliminate a variable by the "addition method," we need to have the variable on the same side of the equal sign with opposite coefficients. Or, we need to have the variable on opposite sides of the equal sign with the same coefficient.

Both of the x-variables are on the left side, so we need opposite coefficients. We can get that by multiplying the first equation by -0.08, or by multiplying the second equation by -12.5. We judge the first of these choices to be easier.

The y-variables are on opposite sides of the equal sign, so we need equal coefficients. We can get that by multiplying the first equation by 0.02, or the second equation by 50.

solution

We choose to multiply the first equation by 0.02, so we can eliminate the y-variable. Here is the result of doing that, then adding the results

  (0.02)(x) +(0.08x +0.02y) = (0.02)(y +1300) +(350)

  0.10x +0.02y = 0.02y +376 . . . . . eliminate parentheses

  0.10x = 376 . . . . . . . . . subtract 0.02y from both sides. y is eliminated

  x = 3760 . . . . . . . . . divide by 0.10

  y = x -1300 = 2460

__

The amount invested at 8% was $3760; the amount invested at 2% was $2460.

_____

Additional comment

We chose to eliminate y for a couple of reasons. x is the amount at the higher rate. We have found that solving for the higher-rate amount usually works best for preventing errors. The other reason is that multiplying by 0.02 results in smaller numbers, which we consider easier to deal with.

Had we multiplied by -0.08 to eliminate x, we would have ...

  -0.08(x) +(0.08x +0.02y) = -0.08(y +1300) +(350)

  0.02y = -0.08y +246

We judge -0.08(1300) +350 harder to calculate mentally, than 0.02(1300) +350.

  0.10y = 246

  y = 2460; x = 2460+1300 = 3760

Comparing Fractions Choose the correct symbol to compare the fractions please see attached image

Answers

Answer:

>

Step-by-step explanation:

Given:      [tex]\frac{1}{2} ? \frac{1}{3}[/tex]

Want:       Which fraction is larger?

To identify the size of a fraction, converting it into a fraction is easier

[tex]\frac{1}{2}[/tex] = 0.5

[tex]\frac{1}{3}[/tex] = 0.33

where 0.5 is greater than 0.33

Choices:

< is less than symbol

> is greater than symbol

= is equal to

[tex]\frac{1}{2} > \frac{1}{3}[/tex]

Learn more about the Fractions at https://brainly.com/question/280013

Visual Representation:

A teacher asked her students how many pets they own. Here
are the results:
0, 0, 0, 1, 1, 1, 1, 2, 2, 2, 3, 3, 4, 4, 8
On a piece of paper, draw a dot plot to represent the data.
Then determine which answer choice matches the dot plot
you drew.

Answers

Step-by-step explanation:

the answer is (c)

because 0 is 3 in number,

1 is 4 in number,

2 is 3in number

3 is 2 in number,

4 is 2 in number,

no 5,6,7

8 is 1 in number

and no 9

The dot plot ( C )is solved and each dot represents one student's response

What is a dot plot?

A dot plot, also known as a dot chart or dot graph, is a type of data visualization that displays the frequency of values in a dataset. It is a simple graphing technique that uses dots to represent data points on a horizontal or vertical axis.

Each data point in the dataset is represented by a dot placed above the corresponding value on the axis. If there are multiple data points with the same value, the dots are stacked vertically above that value. The resulting plot shows the distribution of data points along the axis, with the height of the stack of dots representing the frequency of each value.

Given data ,

Let the data of the number of pets owned by student be represented as A

Now , the value of A = { 0, 0, 0, 1, 1, 1, 1, 2, 2, 2, 3, 3, 4, 4, 8 }

The frequency of 0 = 3

The frequency of 1 = 4

The frequency of 2 = 3

The frequency of 3 = 2

The frequency of 4 = 2

The frequency of 8 = 1

In this dot plot, each dot represents one student's response. The number on the left indicates how many pets the students reported owning, and the dots are arranged vertically above that number.

For example, three students reported owning 2 pets, so there are three dots arranged above the number 2. This type of plot is useful for displaying the frequency of different values in a dataset.

Hence , the dot plot is solved

To learn more about dot plot click :

https://brainly.com/question/30986825

#SPJ5

Determine the number of zeros of the polynomial function. f(x) = x^4 − 6x

Answers

The factor of the function will be x and (x³ – 6). Then the zeroes of the function will be 0 and √6.

What is a factorization?

It is a method for dividing a polynomial into pieces that will be multiplied together. At this moment, the polynomial's value will be zero.

The polynomial function is given below.

f(x) = x⁴ − 6x

Then the factor of the function will be

f(x) = x(x³ – 6)

Then the zeroes of the function will be

x = 0, √6

More about the factorization link is given below.

https://brainly.com/question/6810544

#SPJ1

Lashawn makes scrambled eggs with 3 spoonfuls of salsa for every 2 eggs. Gilberta adds 7 spoonfuls of salsa for every 5 eggs.
Whose scrambled eggs have a stronger salsa taste?

A. Lashawn's eggs
B. Gilberta's eggs
C. The dishes of scrambled eggs have equal salsa tastes

Answers

Lashawn's scrambled eggs have a stronger salsa taste than Gilberta's scrambled eggs. Then the correct option is A.

What is Algebra?

The analysis of mathematical representations is algebra, and the handling of those symbols is logic.

Lashawn makes scrambled eggs with 3 spoonfuls of salsa for every 2 eggs.

Gilberta adds 7 spoonfuls of salsa for every 5 eggs.

Then the scrambled eggs have a stronger salsa taste will be

Lashawn's eggs = 3/2 = 1.5

Gilberta's eggs = 7/5 = 1.4

Lashawn's scrambled eggs have a stronger salsa taste than Gilberta's scrambled eggs.

Then the correct option is A.

More about the Algebra link is given below.

https://brainly.com/question/953809

#SPJ1

At the city museum child admission is $6.10 and adult admission is $9.20. On Wednesday, 144 tickets were sold for a total sales of 1083.00. How many adult tickets were sold that day

Answers

They sold over 500 tickets

Which of the numbers below are whole numbers?
A. 0
B. 0.388
C. 2454
D. 602.49
E. 996
F. 719557

Answers

Answer:

a whole number is a number with no decmils other than .0000 and is not a fraction where the top and bottom are equal

whole: a,c,e&f

not:b&d

The answers are A, C, E & F.

Whole numbers are numbers that don’t have any decimal points aside from .0000 & are not a fraction in some sort of way.

A recipe that makes 18 chocolate chip cookies calls for 2.5 cups of flour. How many cookies can you make if you have 3.75 cups of flour (and enough of the other ingredients)?

Answers

Answer:

27

Step-by-step explanation:

3.75/2.5x18=27

pls someone help me asap :)

Answers

Answer:

right angle

acute

acute

obtuse

obtuse

Step-by-step explanation:

Since angle <ABC is a straight line, the angle is 180 degrees. This means all the angles in between should also add up to 180 degrees. Using this knowledge you can solve for the value of angle <FBE which is going to be equal to 180 - (40 + 25 + 50) = 65.

< DBF = <DBE + <EBF

< DBF = 25 + 65 = 90 (right angle)

<EBD = 25 (given) (acute)

<DBC = 50 (given) (acute)

<ABE = <ABF + <FBE

<ABE = 40 + 65

<ABE = 105 (obtuse)

<CBF = <CBD + <DBE + <EBF

<CBF = 50 + 25 + 65

<CBF = 140 (obtuse)

If $4500 is invested at 10% annual interest, which is compounded continuously, what is the account balance after 3 years, assuming no additional deposits or withdrawals are made?

Answers

Answer:

$6074.36

Step-by-step explanation:

The formula for continuous compounding is

F = Pe^(rt)

F = $4500 × e^(0.1 × 3)

F = $6074.36

Answer: $6074.36

can you help me to solve this

Answers

Answer:

Is the question even complete ?

Step-by-step explanation:

please help me. 50 pts
Which quantity is multiplied by pi () in the formula for the area of a circle?
A. d
OB. 2
O c. d
D. r

Answers

Option: B

The area of a circle is pi times the radius squared (A = π r²).

thanks to you all for you help

Answers

Answer:yw

Step-by-step explanation:

which law would you use to simplify the expression...​

Answers

Answer:

Power of a quotient

A sphere has a radius of 24 centimeters. What is its volume

Answers

Answer:

57905.8

Step-by-step explanation:

Use the formula:
V = 4/3 * πr³

Plug in:

V = 4/3 * 24³π

= 4/3 * 13824π

= 18432π

= 57905.83579cm³

Rounded to nearest tenth:

57905.8

Answer: 18,432

Step-by-step explanation: ANSWER ON EDMENTUM/PLUTO

Which one is it I need to finish this page fast

Answers

Answer:

E; 10% off %20 sweater with $6 shipping

Step-by-step explanation:

Similar to what I said last time,

this time is with percentages,

How to calculate percent off?

Divide the number by 100 (move the decimal place two places to the left).

Multiply this new number by the percentage you want to take off.

Subtract the number from step 2 from the original number. This is your percent off number.
________________________________________________________
A. 35% off a $35 sweater with 4 dollar shipping is basically
  0.35 x 35 = 12.25 and 35-12.25 = $22.75 + the $4 shipping fee (if no tax)
    which the total would be $26.75


B
. 20% off $35 is
   0.2x35=7 and 35-7 = 28 so your total is $28


C.
30% off $30 with $6 for shipping is

   0.3x30=9 and 30-9 is $21 plus the shipping fee
   $27


D.
 $20 sweater with $5 shipping is $25 no discount involved :D


E.
10% off %20 sweater with $6 shipping is

    0.1x20 is 2 which 20 - 2 = 18 and plus shipping fee 18+6 is $24
________________________________________________________
Hope this helped again.

Help with this math problem

Answers

Answer:

The minimum cost of the X-ray machines is 12,197dollars.

Step-by-step explanation:

First, we check whether it’s a Quadratic Equation or not

The general form of the Quadratic equation [tex]f(x) =[/tex] [tex]ax^{2} + bx + c[/tex]

Let’s compare it with the given equation, we get

a = 1

b = -520

c = 79,79

Hence, it’s a quadratic equation.

Now, we will use the VERTEX FORMULA as we are asked to find the ‘minimum’ unit cost.

X = [tex]\frac{-b}\[2a[/tex]  =  [tex]\frac{-(-520)}\[2(1)[/tex] = [tex]260[/tex]

So , number of X-ray machines are = 260 (value of x)

To find the minimum unit cost, plug the value of x into the given equation, and we get

[tex]f(260) = (260)^{2} -520(260) + 79,797[/tex]

[tex]=12,197[/tex]

Learn more about quadratic equation here: https://brainly.com/question/23511870

#SPJ10

The points (-15, r) and (-3, 16) lie on a line with slope 5/4. Find the missing coordinate r.

Answers

Answer:

r = 1

Step-by-step explanation:

we know that :

[tex]Slope=\frac{y_{B}-y_{A}}{x_{B}-x_A}[/tex]

Then

[tex]\frac{5}{4} =\frac{16-r}{-3-(-15)}[/tex]

Then

[tex]\frac{5}{4} =\frac{16-r}{12}[/tex]

Then

5×12 = 4×(16-r)

Then

60 = 64 - 4r

Then

4r = 64 - 60

Then

4r = 4

Then

r = 1

calculus piecewise function. ​

Answers

Part A

The notation [tex]\lim_{x \to 2^{+}}f(x)[/tex] means that we're approaching x = 2 from the right hand side (aka positive side). This is known as a right hand limit.

So we could start at say x = 2.5 and get closer to 2 by getting to x = 2.4 then to x = 2.3 then 2.2, 2.1, 2.01, 2.001, etc

We don't actually arrive at x = 2 itself. We simply move closer and closer.

Since we're on the positive or right hand side of 2, this means we go with the rule involving x > 2

Therefore f(x) = (x/2) + 1

Plug in x = 2 to find that...

f(x) = (x/2) + 1

f(2) = (2/2) + 1

f(2) = 2

This shows [tex]\lim_{x \to 2^{+}}f(x) = 2[/tex]

Then for the left hand limit [tex]\lim_{x \to 2^{-}}f(x)[/tex], we'll involve x < 2 and we go for the first piece. So,

f(x) = 3-x

f(2) = 3-2

f(2) = 1

Therefore, [tex]\lim_{x \to 2^{-}}f(x) = 1[/tex]

===============================================================

Part B

Because [tex]\lim_{x \to 2^{+}}f(x) \ne \lim_{x \to 2^{-}}f(x)[/tex] this means that the limit [tex]\lim_{x \to 2}f(x)[/tex] does not exist.

If you are a visual learner, check out the graph below of the piecewise function. Notice the gap or disconnect at x = 2. This can be thought of as two roads that are disconnected. There's no way for a car to go from one road to the other. Because of this disconnect, the limit doesn't exist at x = 2.

===============================================================

Part C

You'll follow the same type of steps shown in part A.

However, keep in mind that x = 4 is above x = 2, so we'll deal with x > 2 only.

So you'd only involve the second piece f(x) = (x/2) + 1

You should find that f(4) = 3, and that both left and right hand limits equal this value. The left and right hand limits approach the same y value. The limit does exist here. There are no gaps to worry about when x = 4.

===============================================================

Part D

As mentioned earlier, since [tex]\lim_{x \to 4^{+}}f(x) = \lim_{x \to 4^{-}}f(x) = 3[/tex], this means the limit [tex]\lim_{x \to 4}f(x)[/tex] does exist and it's equal to 3.

As x gets closer and closer to 4, the y values are approaching 3. This applies to both directions.

the equation of the line that goes through (3, 0) and (6, 2) in slope-intercept form???

Answers

the slope intercept form of the given points is  y = 2/3 (x-3).

What is Straight Line?

A straight line is an endless one-dimensional figure that has no width. It is a combination of endless points joined on both sides of a point. A straight line does not have any curve in it.

Here, given passing points

(3, 0) and (6, 2)

We know that,

    (y - y₁) = (y₂-y₁)/(x₂-x₁) (x - x₁)

  (y - 0) = (2-0)/(6-3) (x-3)

    y = 2/3 (x-3)

Thus, the slope intercept form of the given points is  y = 2/3 (x-3).

Learn more about Straight line from:

https://brainly.com/question/27560536

#SPJ1

On a number line, 6.87 would be located
6.87
A. To the right of 6.81
B. Between 6 and 6.5
C. Between 6.5 and 7
D. To the left of 6.81
Check all that apply.

Answers

A and C are the answers

There are 5 white balls. 8 red balls, 7 yellow balls, and 4 green balls in a container. A ball is chosen randomly. Find the probability of the following events.

1.What is the probability of choosing

green?
Answers:________ work space:______

2. What is the probability of choosing

red?
Answers:________ work space:______

3. What is the probability of choosing

neither white nor green?
Answers:________ work space:_______

4. . What is the probability of choosing

other than yellow?
Answers:________ work space:_______

5. What is the probability of choosing

purple?
Answer:_________ work space:_______​

Answers

Given :

White balls = 5 Red balls = 8 Yellow balls = 7Green balls = 4

Answer 1 :-

We know that,

P (E) = No. of favourable outcomes / Total number of outcomes.

>> Total no. of outcomes = 5 + 8 + 7 + 4

>> Total no. of outcomes = 24

As there are 4 green balls. Therefore,

P (E) = 4 / 24P (E) = 2 / 12 P (E) = 1 / 6

Answer 2 :-

We know that,

P (E) = No. of favourable outcomes / Total number of outcomes.

>> Total no. of outcomes = 5 + 8 + 7 + 4

>> Total no. of outcomes = 24

As there are 8 red balls. Therefore,

P (E) = 8 / 24 P (E) = 4 / 12 P (E) = 2 / 6 P (E) = 1 / 3

Answer 3 :-

We know that,

P (E) = No. of favourable outcomes / Total number of outcomes.

>> Total no. of outcomes = 5 + 8 + 7 + 4

>> Total no. of outcomes = 24

As there are 15 balls that are neither green nor white. (Red balls+ yellow balls)

P (E) = 15 / 24 P (E) = 5 / 8

Determine the values of x and y.

Answers

Answer: x = 23, y = 40

Step-by-step explanation:

By the same-side interior angles theorem, y+30 and 110 are supplementary.

110 + y + 30 = 180 [supplementary angles add to 180 degrees]y + 140 = 180 [combine like terms]y = 40 [subtract 140 from both sides]

We know that the 3x + 1 angle and y + 30 are also congruent, so:

3x + 1 = y + 30 [congruent angles have equal measure]3x + 1 = 40 + 30 [substitute y = 40]3x + 1 = 70 [combine like terms]3x = 69 [subtract 1 from both sides]x = 23 [divide both sides by 3]

Construct the truth table and determine the truth value of the following compound statement.
a)p⟾¬(p ʌ r)
b) (q ʌ r) ⟾(p ⇔ q)​

Answers

See the attached truth tables.

• A ∧ B is true only when both A and B are true

• A ⇒ B is true only when both A and B are true, or A is false. This logically equivalent to ¬A ∨ B

• ¬A is true only when A is false

• A ⇔ B is true only when both A ⇒ B and B ⇒ A are true. Equivalently, (¬A ∨ B) ∧ (¬B ∨ A)

HURRY Given f(x) = - 3/4x + 2, find ƒ(16). The solution is

Answers

Answer: Read attachment.

Step-by-step explanation: Attached is my work. Hope it helps!

If my answer was helpful, please mark as brainliest. Or not, it's fine.

Answer:

[tex]\fbox {f(16) = -10}[/tex]

Step-by-step explanation:

Given :

f(x) = -3/4x + 2

Substitute x = 16 :

f(16) = -3/4(16) + 2

f(16) = -12 + 2

f(16) = -10

Other Questions
can teachers tell if you use Brainly How did World War II change the global balance of power?OA. It helped Asia become the central world power because of thewar's limited effect on the region.B. It resulted in the United States and the Soviet Union becoming theworld's two dominant powers.C. It allowed the Axis powers to emerge from the war as the world'smost powerful economies.OD. It spread global power almost equally among the countries thatjoined the United Nations. What happened as southern Democrats gained power? a die is tossed. find the odds against rolling a number greater than 1 Relating to ideas, customs, and social behaviors of a society is the definition of what? Which Greek geometer founded a philosophical society that devoted itself to the study of mathematics?A. ThalesB. PythagorasC. EuclidD. Hippocrates Why is syntax important in a piece of writing In what table does y vary directly with x? is about and DC are parallel, which angles are congruent to D 3? Which is the best description of authorized shares? A US senator is sponsoring a bill to protect an endangered species of turtle. The bill has been narrowly approved by theSenate. A similar bill has been approved by the House of Representatives by a greater margin than the Senate bill.What is the next step that will lead to one version of the two bills being signed into law?OA. The Senate bill will become law, because the Senate is the more powerful of the two congressional chambers.OB. The House bill will become law, because it won a higher percentage of votes than the Senate bill.OC. The Senate bill and the House bill will go to a joint committee of lawmakers, who will work out a compromise bill.Both bills will go to the president, who will choose the one he or she thinks is better and sign that one into law.D. Could use help asap please !!!!!!!!!!!!!!!!!!!!!!!!!!!!!!!!!!!!!!!!! Select the correct answer.Which expression is equivalent to 48x5, if x > 0?OA. 12x3xOB. 4x3OC. 4x3xOD. 12x Ella ________abgada. According to supreme court custom, a case is granted a writ of certiorari when at least how many justices vote to do so?. Read the passage from Lord of the Flies by William Golding.Jack drew up his legs, clasped his knees, and frowned in an effort to attain clarity."All the samein the forest. I mean when youre hunting, not when youre getting fruit, of course, but when youre on your ownHe paused for a moment, not sure if Ralph would take him seriously."Go on."If youre hunting sometimes you catch yourself feeling as if He flushed suddenly. "Theres nothing in it of course. Just a feeling. But you can feel as if youre not hunting, butbeing hunted, as if somethings behind you all the time in the jungle. . . ."The best thing we can do is get ourselves rescued.Jack had to think for a moment before he could remember what rescue was."Rescue? Yes, of course! All the same, Id like to catch a pig first He snatched up his spear and dashed it into the ground. The opaque, mad look came into his eyes again. Ralph looked at him critically through his tangle of fair hair.How are the universal themes "the relationship between violence and human nature and "the relationship between civilization and nature best developed in this passage?Golding uses conflict to highlight why the boys have different priorities.Golding uses description to emphasize how the island has influenced Jack.Golding uses internal thoughts to describe Jacks confusion.Golding uses dialogue to explain why the boys are afraid. What is Kingdom Monera? The coordinates of the vertices of ABC are A(2,2), B(5,3), and C(4,1). Identify the perimeter of ABC. Round each side length to the nearest tenth before adding. List and describe four services that comprise IT infrastructure, beyond physical devices and software applications.